Difference between revisions of "1990 AIME Problems/Problem 3"

m
Line 1: Line 1:
{{empty}}
 
 
== Problem ==
 
== Problem ==
 +
Let <math>P_1^{}</math> be a regular <math>n~\mbox{gon}</math> and <math>P_2^{}</math> be a regular <math>s~\mbox{gon}</math> <math>(r\geq s\geq 3)</math> such that each interior angle of <math>P_1^{}</math> is <math>\frac{59}{58}</math> as large as each interior angle of <math>P_2^{}</math>. What's the largest possible value of <math>s_{}^{}</math>?
  
 
== Solution ==
 
== Solution ==
 
{{solution}}
 
{{solution}}
 +
 
== See also ==
 
== See also ==
* [[1990 AIME Problems]]
+
{{AIME box|year=1990|num-b=2|num-a=4}}

Revision as of 01:20, 2 March 2007

Problem

Let $P_1^{}$ be a regular $n~\mbox{gon}$ and $P_2^{}$ be a regular $s~\mbox{gon}$ $(r\geq s\geq 3)$ such that each interior angle of $P_1^{}$ is $\frac{59}{58}$ as large as each interior angle of $P_2^{}$. What's the largest possible value of $s_{}^{}$?

Solution

This problem needs a solution. If you have a solution for it, please help us out by adding it.

See also

1990 AIME (ProblemsAnswer KeyResources)
Preceded by
Problem 2
Followed by
Problem 4
1 2 3 4 5 6 7 8 9 10 11 12 13 14 15
All AIME Problems and Solutions